Aug 27

今天介绍一个用多项式一致逼近函数的定理。

定理:令$f$为定义于区间$[0,1]$上的连续函数,对任意$\varepsilon>0$存在多项式函数$p$使得对所有$x\in[0,1]$不等式
\[ |f(x)-p(x)|<\varepsilon \]
成立。
本定理证明依赖于以下几件事:
(a)定义于闭区间上的连续函数是一致连续的:对任意$\varepsilon>0$,存在$\delta>0$,使得对任意的$x,y$满足$|x-y|<\delta$,就有$|f(x)-f(y)|<\varepsilon $
(b)定义于闭区间上的连续函数是有界的:存在$M>0$使得$|f(x)|<M$。
(c)恒等式
\[ \sum_{k=0}^{n}\left(\frac{k}{n}-x\right)^{2}C_{n}^{k}x^k(1-x)^{n-k}=\frac{x(1-x)}{n} \]

————————————————————————————————————————————————————————————
为了证明(c),令$f(x)=(x+t)^n$,由牛顿二项式定理得
\[ f(x)=\sum_{k=0}^{n}C_{n}^{k}x^{k}t^{n-k} \]
两边对$x$求导,得到
\[ n(x+t)^{n-1}=f'(x)=\sum_{k=0}^{n}C_{n}^{k}kx^{k-1}t^{n-k} \]

\[ x(x+t)^{n-1}=\sum_{k=0}^{n}\frac{k}{n}C_{n}^{k}x^{k}t^{n-k} \]
令$g(x)=x(x+t)^{n-1}$.则
\[ g'(x)=(x+t)^{n-1}+(n-1)x(x+t)^{n-2}=\sum_{k=0}^{n}\frac{k}{n}\cdot C_{n}^{k}kx^{k-1}t^{n-k} \]
\[ \Rightarrow\qquad \frac{x}{n}(x+t)^{n-1}+\frac{x^2}{n}(n-1)(x+t)^{n-2}=\sum_{k=0}^{n}\frac{k^2}{n^2}C_{n}^{k}x^{k}t^{n-k} \]
将$t=1-x$代入得到
\begin{equation}
 1=\sum_{k=0}^{n}C_{n}^{k}x^{k}(1-x)^{n-k}
\end{equation}
\begin{equation}
 x=\sum_{k=0}^{n}\frac{k}{n}C_{n}^{k}x^{k}(1-x)^{n-k}
\end{equation}
\begin{equation}
\frac{x}{n}+\frac{x^2}{n}(n-1)=\sum_{k=0}^{n}\frac{k^2}{n^2}C_{n}^{k}x^{k}(1-x)^{n-k} 
\end{equation}
于是
\begin{align*}
  &\sum_{k=0}^{n}\left(\frac{k}{n}-x\right)^{2}C_{n}^{k}x^k(1-x)^{n-k}\\
  &=\sum_{k=0}^{n}\frac{k^2}{n^2}C_{n}^{k}x^{k}(1-x)^{n-k}-2x\sum_{n=0}^{n}\frac{k}{n}C_{n}^{k}x^{k}(1-x)^{n-k}+x^2\sum_{k=0}^{n}C_{n}^{k}x^{k}(1-x)^{n-k}\\
  &=\frac{x}{n}+\frac{x^2}{n}(n-1)-2x^2+x^2\\
  &=\frac{x(1-x)}{n}
\end{align*}
\[\Leftrightarrow\qquad \sum_{k=0}^{n}\left(k-nx\right)^{2}C_{n}^{k}x^k(1-x)^{n-k}=nx(1-x) \]

——————————————————————————————————————————————————————————————
定理证明
令$\varepsilon>0$,存在$\delta>0$,使得当$x,y\in[0,1]$,$|x-y|<\delta$,不等式$|f(x)-f(y)|<\frac{\varepsilon}{2}$成立。同时可以选取$M>0$,使得$|f(x)|<M$.考虑$f(x)$的Bernstein 多项式
\[ \boxed{B_{n}(x)=\sum_{k=0}^{n}f\left(\frac{k}{n}\right)C_{n}^{k}x^{k}(1-x)^{n-k}\qquad (n=1,2,\cdots)} \]
我们将证明:若$n>\max\bigg\{\frac{1}{\delta^{4}},\left(\frac{4M}{\varepsilon}\right)^{2}\bigg\}$,则所有$x\in[0,1]$都满足$|f(x)-B_{n}(x)|<\varepsilon$.证明如下
\[ f(x)-B_{n}(x)=\sum_{k=0}^{n}\left(f(x)-f\left(\frac{k}{n}\right)\right)C_{n}^{k}x^{k}(1-x)^{n-k}=\sum_{(a)}+\sum_{(b)} \]
其中$\sum_{(a)}$表示对满足$\left|\frac{k}{n}-x\right|<\frac{1}{n^{\frac{1}{4}}}$求和,而$\sum_{(b)}$表示对满足$\left|\frac{k}{n}-x\right|\geq \frac{1}{n^{\frac{1}{4}}}$求和。
若$\left|\frac{k}{n}-x\right|\geq \frac{1}{n^{\frac{1}{4}}}$,则$|k-nx|\geq n^{\frac{3}{4}}$,故$(k-nx)^{2}\geq n^{\frac{3}{2}}$.由此
\begin{align*}
\left|\sum_{(b)}\right|&\leq \sum_{(b)}\left[|f(x)|+\left|f\left(\frac{k}{n}\right)\right|\right]C_{n}^{k}x^{k}(1-x)^{n-k}\\
&\leq 2M\sum_{(b)}C_{n}^{k}x^{k}(1-x)^{n-k}\\
&\leq \frac{2M}{n^{\frac{3}{2}}}\sum_{(b)}C_{n}^{k}(k-nx)^2x^{k}(1-x)^{n-k}\\
&\leq \frac{2M}{n^{\frac{3}{2}}}nx(1-x)\\
&\leq \frac{2M}{n^{\frac{1}{2}}}<\frac{\varepsilon}{2}
 \end{align*}
 若$\left|\frac{k}{n}-x\right|<\frac{1}{n^{\frac{1}{4}}}$,则$\left|\frac{k}{n}-x\right|<\delta$,故$\left|f\left(\frac{k}{n}\right)-f(x)\right|<\frac{\varepsilon}{2}$,由此
 \[ \left|\sum_{a}\right|\leq\sum_{a}\left|f(x)-f\left(\frac{k}{n}\right)\right|C_{n}^{k}x^{k}(1-x)^{n-k}<\frac{\varepsilon}{2} \]
 因此,马上有
 \[ |f(x)-B_{n}(x)|<\varepsilon \]
 $B_{i}^{n}(t)$称为Bernstein基底
 \[ B_{i}^{n}=C_{n}^{i}t^{i}(1-t)^{n-i} \]

Aug 26

问题1
设函数$f$在$[0,+\infty)$上一致连续,且$\forall x\in[0,1]$有
\[ \lim_{n\to+\infty}f(x+n)=0 \qquad (n\in \mathbf{N}) \]
证明:
\[ \lim_{x\to+\infty}f(x)=0 \]
证明
由于$f$在$[0,+\infty)$上一致连续,故对$\forall \varepsilon>0$,$\exists \delta>0$,当$x,y$ 满足$ |x-y|<\delta$时,有
\[ |f(x)-f(y)|<\varepsilon \]
这样,我们可以把区间$[0,1]$分成$m$等份,记座$\Delta_{1},\Delta_{2},\cdots,\Delta_{n} $分点为$x_{0}=0<x_{1}=\frac{1}{m}<x_{2}=\frac{2}{m}<\cdots<1=\frac{m}{m}=x_{m} $,其中$m$满足$\frac{1}{m}<\delta$.
那么,对于这$m+1$个分点$x_{i},(i=0,1,\cdots,m)$,由于
\[ \lim_{n\to+\infty}f(x+n)=0\]
则有
\[  \lim_{n\to+\infty}f(x_{i}+n)=0 \qquad (i=0,1,\cdots,m)\]
换一种说法就是对$\forall \varepsilon>0$,及每一个$x_{i} (i=0,1,\cdots,m)$,存在$ N_{i}\in \mathbf{N},(i=0,1,\cdots,m)$, 当$n> N_{i}$时有
\[ |f(x_{i}+n)|<\varepsilon \]
这样,我们可以取$N=\max\{N_{0},N_{1},\cdots N_{m}\}$,于是当$n>N$时,有
\[ |f(x_{i}+n)|<\varepsilon\qquad (i=0,1,\cdots,m) \]
这样,对$ \forall x>N+1$,$x=[x]+t_{0}$,其中$t_{0}\in [0,1) $,这样既有$[x]>N$,又有$t_{0}$落入某个$\Delta_{i}$,不妨设$t_{0}$落入第$k$个区间,显然有$|t_{0}-x_{k}|<\frac{1}{m}<\delta$,这样根据一致连续性有
\[ |f(x_{k}+[x])-f(x)|=|f(x_{k}+[x])-f([x]+t_{0})|<\varepsilon \]
而由于$[x]>N$,则又有
\[ |f(x_{k}+[x])|<\varepsilon\]
马上看到
\[ |f(x)|\leq |f(x)-f(x_{k}+[x])|+|f(x_{k}+[x])|<2\varepsilon\]
由此知
\[ \lim_{x\to+\infty}f(x)=0 \]

——————————————————————————————————————————————————————————————————

问题2.
设函数$f$在$[0,+\infty)$上连续,$\forall \alpha>0$,有$ \displaystyle \lim_{n\to+\infty}f(n\alpha)=0$. 证明:
\[ \lim_{x\to+\infty}f(x)=0 \]
这个问题显然比上面的问题要复杂不少,去掉一致连续性条件弱了好多。
证明
$\forall\varepsilon>0$,显然$ A_{n}=\{\alpha||f(n\alpha)|\leq \varepsilon\}$是个闭集,(由$f$的连续性不难验证,$A_{n}^{c}$是开集),$\displaystyle B_{k}=\bigcap_{n>k}{A_{n}}$也为闭集。
$\forall x\in(0,+\infty)$,由于$\displaystyle \lim_{n\to +\infty}f(nx)=0$,知,$\exists k\in \mathbf{N}$, 当$n>k$时$|f(nx)|<\varepsilon$,于是$x\in B_{k}$,因此
\[ (0,+\infty)=\bigcup_{k=1}^{\infty} B_{k} \]
由著名的Baire定理知,$ \exists [a,b]\subset B_{k_{0}}$.取$l\geq \max\bigg\{k_{0}+1,\frac{a}{b-a}\bigg\}$,当$n\geq l$时,有$n(b-a)\geq a$从而$nb\geq (n+1)a$.于是,对$\forall x>la,$即 $\displaystyle x\in[la,+\infty)\subset \bigcup_{n>k_{0}}[na,nb]$,存在某个$n>k_{0}$ 使得$x\in[na,nb]$,从而存在$\displaystyle \alpha\in[a,b]\subset B_{k_{0}}=\bigcap_{n>k_{0}}A_{n}$,$\alpha\in A_{n},n>k_{0}$,使得$x=n\alpha$,所以
\[ |f(x)|=|f(n\alpha)|\leq \varepsilon \]

\[ \lim_{x\to+\infty}f(x)= 0 \]

著名的Baire定理是这么说的:设$E,E_{m}\in \mathbf{R^{n}}$,$\displaystyle E=\bigcup_{m\in\Gamma}E_{m}$, 其中$\Gamma$为至多可数集。$E_{m}$的内点集$ \text{int}E_{m}=\varnothing$,$E_{m}$为闭集,$m\in\Gamma$,则$\text{int}E=\varnothing$.对刚刚的问题,我们知道若每一个$B_{k}$都没有内点,那么会得到$(0,+\infty)$没内点,显然是个矛盾,故存在某个$B_{k_{0}}$有内点,显然就存在一个$[a,b]\subset B_{k_{0}}$.

Jun 22

证明
\[ \lim_{n\to\infty}{\int_{0}^{\frac{\pi}{2}}{\sin{t^{n}}dt}}=0 \]



\[ I=\int_{0}^{\frac{\pi}{2}}{\sin{t^{n}}dt}=\frac{1}{n}\cdot\int_{0}^{\left(\frac{\pi}{2}\right)^{n}}{y^{\frac{1}{n}-1}\sin{y}dy}\qquad (y=t^{n}) \]

\[ \Gamma\left(1-\frac{1}{n}\right)=\int_{0}^{\infty}{u^{-\frac{1}{n}}e^{-u}du}=\int_{0}^{\infty}{y^{1-\frac{1}{n}}\cdot x^{-\frac{1}{n}}e^{-xy}dx} \]
\begin{align*}
 I&=\frac{1}{n\Gamma\left(1-\frac{1}{n}\right)}\int_{0}^{\left(\frac{\pi}{2}\right)^{n}}{\left(\int_{0}^{+\infty}{x^{-\frac{1}{n}}e^{-xy}dx}  \right)\cdot\sin{y}dy}\\
&=\frac{1}{n\Gamma\left(1-\frac{1}{n}\right)}\int_{0}^{+\infty}{x^{-\frac{1}{n}}\cdot\left(\int_{0}^{\left(\frac{\pi}{2}\right)^{n}}{e^{-xy}\sin{y}dy}  \right)dx}\\
&=\frac{1}{n\Gamma\left(1-\frac{1}{n}\right)}\int_{0}^{+\infty}{\frac{x^{-\frac{1}{n}}}{1+x^2}dx}-\frac{1}{n\Gamma\left(1-\frac{1}{n}\right)}\int_{0}^{\infty}{\frac{x^{-\frac{1}{n}}\left[\cos\left(\frac{\pi}{2}\right)^{n}+\left(\frac{\pi}{2}\right)^{n}\sin\left(\frac{\pi}{2}\right)^{n}  \right] }{(1+x^2)e^{x\left(\frac{\pi}{2}\right)^{n}}}dx}\\
&=I_{1}-I_{2}
\end{align*}
而我们知道
\[ \int_{0}^{+\infty}{\frac{t^{a-1}}{1+t}dt}=\frac{\pi}{\sin{\pi a}}\qquad  (\text{Euler}) \]
\[ \Rightarrow \int_{0}^{+\infty}{\frac{x^{-\frac{1}{n}}}{1+x^2}dx}=\frac{\pi}{2\cos{\frac{1}{2n}}}\]
\[ \Rightarrow \lim_{n\to\infty}{I_{1}}=0 \]
\[ I_{2}=\frac{1}{n\Gamma\left(1-\frac{1}{n}\right)}\cdot\left(\int_{0}^{1}+\int_{1}^{+\infty}\right)=S_{1}+S_{2}\]
\begin{align*}
|S_{1}|&=\frac{1}{n\Gamma\left(1-\frac{1}{n}\right)}\left|\int_{0}^{1}{\frac{x^{-\frac{1}{n}}\left[\cos\left(\frac{\pi}{2}\right)^{n}+\left(\frac{\pi}{2}\right)^{n}\sin\left(\frac{\pi}{2}\right)^{n}  \right] }{(1+x^2)e^{x\left(\frac{\pi}{2}\right)^{n}}}dx} \right|\\
&\leq\frac{1}{n\Gamma\left(1-\frac{1}{n}\right)}\int_{0}^{1}{\frac{[\left(\frac{\pi}{2}\right)^{n}+1]x^{-\frac{1}{n}}}{e^{x\left(\frac{\pi}{2}\right)^{n}}}dx}\\
&= \frac{1}{n\Gamma\left(1-\frac{1}{n}\right)}\cdot \frac{\left(\frac{\pi}{2}\right)^{n}+1}{\left(\frac{\pi}{2}\right)^{n-1}}\int_{0}^{\left(\frac{\pi}{2}\right)^{n}}{z^{-\frac{1}{n}}e^{-z}dz}\\
&\leq \frac{1}{n\Gamma\left(1-\frac{1}{n}\right)}\cdot \frac{\left(\frac{\pi}{2}\right)^{n}+1}{\left(\frac{\pi}{2}\right)^{n-1}}\int_{0}^{+\infty}{z^{-\frac{1}{n}}e^{-z}dz}\\
&=\frac{1}{n}\cdot\left[\left( \frac{\pi}{2}\right)+\left( \frac{\pi}{2}\right)^{1-n}  \right]\to 0
\end{align*}
\begin{align*}
 |S_{2}|&= \frac{1}{n\Gamma\left(1-\frac{1}{n}\right)}\left|\int_{1}^{\infty}{\frac{x^{-\frac{1}{n}}\left[\cos\left(\frac{\pi}{2}\right)^{n}+\left(\frac{\pi}{2}\right)^{n}\sin\left(\frac{\pi}{2}\right)^{n}  \right] }{(1+x^2)e^{x\left(\frac{\pi}{2}\right)^{n}}}dx}\right|\\
&\leq \frac{1}{n\Gamma\left(1-\frac{1}{n}\right)}\cdot \int_{1}^{+\infty}{\frac{x^{-\frac{1}{n}}}{1+x^2}dx}\\
&\leq \frac{\pi}{2n\Gamma\left(1-\frac{1}{n}\right)}\to 0
\end{align*}

\[ \lim_{n\to\infty}{\int_{0}^{\frac{\pi}{2}}{\sin{t^{n}}dt}}=0 \]

——————————————————————————————————————————
顺便计算
\begin{align*}
\int_0^\infty \sin \left( x^n\right)dx &= \frac{1}{n}\int_0^\infty x^{\frac{1}{n}-1} \sin(x) \ dx \quad (x^n \mapsto x) \\
&= \frac{1}{n \Gamma \left( 1-\frac{1}{n}\right)}\int_0^\infty \left(\int_0^\infty u^{-\frac{1}{n}}e^{-xu}du\right) \sin(x) \ dx \\
&= \frac{1}{n \Gamma \left( 1-\frac{1}{n}\right)} \int_0^\infty u^{-\frac{1}{n}} \left( \int_0^\infty e^{-xu}\sin(x) \ dx\right)du \\
&= \frac{1}{n \Gamma \left( 1-\frac{1}{n}\right)} \int_0^\infty \frac{u^{-\frac{1}{n}}}{1+u^2}du \\
&= \frac{1}{n \Gamma \left( 1-\frac{1}{n}\right)} \int_0^{\frac{\pi}{2}}\tan^{-\frac{1}{n}}(\theta) d\theta \quad (u=\tan \theta) \\
&= \frac{1}{n \Gamma \left( 1-\frac{1}{n}\right)}\int_0^{\frac{\pi}{2}}\sin^{-\frac{1}{n}}(\theta) \cos^{\frac{1}{n}}(\theta) d\theta \\
&= \frac{1}{2n \Gamma \left( 1-\frac{1}{n}\right)} \mathrm{B} \left( \frac{1-n}{2},\frac{1+n}{2}\right) \\
&= \frac{1}{2n \Gamma \left( 1-\frac{1}{n}\right)} \Gamma \left( \frac{n-1}{2n}\right)\Gamma \left( \frac{n+1} {2n}\right) \\
&= \frac{\sin \left( \frac{\pi}{n}\right)}{2n\cos \left( \frac{\pi}{2n}\right)}\Gamma \left( \frac{1}{n}\right) \\
&= \frac{1}{n}\sin \left(\frac{\pi }{2n} \right)\Gamma \left( \frac{1}{n}\right)
\end{align*}

——————————————————————————————————————————————

显然,上面的证明弄复杂了,太非主流了。下面给出个主流的证明

对$\forall \varepsilon>0$,存在$ 0<a<\frac{\varepsilon}{4} $,注意到
\begin{align*}
\int_{0}^{\frac{\pi}{2}}{\sin{t^n}dt}&= \int_{0}^{1-a}{\sin{t^n}dt}+ \int_{1-a}^{1+a}{\sin{t^n}dt}+ \int_{1+a}^{\frac{\pi}{2}}{\sin{t^n}dt}\\
&=I_{1}+I_{2}+I_{3}
\end{align*}
由$ |\sin{x}|\leq 1 $知
\[ |I_{2}|<\frac{\varepsilon}{2} \]
对$I_{1}$,有
\[ |I_{1}|\leq \int_{0}^{1-a}{|\sin{t^n}|dt}\leq \int_{0}^{1-a}{t^{n}dt}\leq \frac{(1-a)^{n+1}}{n+1} \]
显然可以找到一个$N_{1}>0$使得$n>N_{1}$时有
\[ |I_{1}|\leq \frac{\varepsilon}{4} \]
而对$I_{3}$
\begin{align*}
 I_{3}&=\int_{1+a}^{\frac{\pi}{2}}{\sin{t^n}dt}\\
&= \int_{1+a}^{\frac{\pi}{2}}{\frac{d(-\cos{t^n})}{nt^{n-1}}}\\
&=\frac{-\cos t^n}{nt^{n-1}}\bigg|_{1+a}^{\frac{\pi}{2}}+\frac{1-n}{n}\cdot \int_{1+a}^{\frac{\pi}{2}}{\frac{\cos{t^{n}}}{t^{n}}dt}\\
&=\frac{\cos{(1+a)^n}-\cos{\left(\frac{\pi}{2}\right)^n}}{n(1+a)^{n-1}}+\frac{1-n}{n}\cdot \int_{1+a}^{\frac{\pi}{2}}{\frac{\cos{t^{n}}}{t^{n}}dt}
\end{align*}
显然存在$N_{2}$使得当$n>N_{2}$时有
\[ |I_{3}|<\frac{\varepsilon}{4} \]
这样,取$N=\max\{N_{1},N_{2}\} $,当$n>N$时,就有
\[ |I|<\varepsilon \]
\[ \Rightarrow \lim_{n\to\infty}{\int_{0}^{\frac{\pi}{2}}{\sin{t^n}}dt}=0 \]

Jun 4

计算
\[ I=\int_{0}^{\pi}{\frac{x\cos{x}}{1+\sin^{2}{x}}dx} \]

(tian_275461)

\[ I=\int_{0}^{\pi}{xd\arctan{(\sin{x})}}=-\int_{0}^{\pi}{\arctan{(\sin{x})}dx}=-2\int_{0}^{\frac{\pi}{2}}{\arctan{(\sin{x})}dx}\]

注意到
\[ \arctan{\left(\frac{\sin{x}}{+\infty}\right)}-\arctan{\left(\frac{\sin{x}}{1}\right)}=-\int_{1}^{+\infty}{\frac{\sin{x}}{y^2+\sin^2{x}}dy}\]

\begin{align*}
I&=-2\int_{0}^{\frac{\pi}{2}}{\int_{1}^{+\infty}{\frac{\sin{x}}{y^2+\sin^2{x}}dy}dx}\\
&=-2\int_{1}^{+\infty}{\int_{0}^{\frac{\pi}{2}}{\frac{\sin{x}}{y^2+\sin^2{x}}dx}dy}\\
&=-2\int_{1}^{+\infty}{\int_{0}^{1}{\frac{1}{y^2+1-t^2}dt}dy}\qquad (t=\cos{x}) \\
&=-\int_{1}^{+\infty}{\frac{1}{\sqrt{y^2+1}}\ln{\left(\frac{\sqrt{y^2+1}+1}{\sqrt{y^2+1}-1}  \right)}dy}\\
&=-\int_{\text{arcsinh}1}^{+\infty}{\ln\left(\frac{\cosh{z}+1}{\cosh{z}-1}\right)dz}\qquad (y=\sinh{z})\\
&=2\int_{\text{arcsinh}1}^{+\infty}{\ln\left(\frac{1-e^{-z}}{1+e^{-z}}\right)dz}\\
&=2\int_{0}^{\sqrt{2}-1}{\frac{\ln{(1-t)}-\ln(1+t)}{t}dt}\qquad (t=e^{-z})\\
&=2\text{Li}_{2}(1-\sqrt{2})-2\text{Li}_{2}(\sqrt{2}-1)
\end{align*}
套用多重对数函数的性质(2)(3)(4)
(2)
\[ \text{Li}_{2}(1-x)+\text{Li}_{2}\left(1-\frac{1}{x}\right)=-\frac{1}{2}\ln^2{x} \]
(3)
\[ \text{Li}_{2}(x)+\text{Li}_{2}(1-x)=\frac{1}{6}\pi^2-\ln{x}\cdot\ln(1-x) \]
(4)
\[ \text{Li}_{2}(-x)-\text{Li}_{2}(1-x)+\frac{1}{2}\text{Li}_{2}(1-x^2)=-\frac{1}{12}\pi^2-\ln{x}\cdot\ln{(x+1)}\]
得到
\begin{align*}
   \text{Li}_{2}(2-\sqrt{2})+ \text{Li}_{2}(-\sqrt{2})&=-\frac{1}{2}\ln^2(\sqrt{2}-1) \qquad (\text{用$x=\sqrt{2}-1$套第2个})\\
    \text{Li}_{2}(2-\sqrt{2})+ \text{Li}_{2}(\sqrt{2}-1)&=\frac{\pi^2}{6}-\ln{(\sqrt{2}-1)}\cdot\ln{(2-\sqrt{2})}\qquad (\text{用$x=\sqrt{2}-1$套第3个})\\
     \text{Li}_{2}(-\sqrt{2})- \text{Li}_{2}(1-\sqrt{2})+\frac{1}{2} \text{Li}_{2}(1-(-\sqrt{2})^2)&=-\frac{\pi^2}{12}-\ln{\sqrt{2}}\cdot\ln{(1+\sqrt{2})}\qquad (\text{用$x=-\sqrt{2}$套第4 个})
\end{align*}
用第2个式子减去第1个式子加上第3个式子,得到
\[  \text{Li}_{2}(\sqrt{2}-1)- \text{Li}_{2}(1-\sqrt{2})=\frac{\pi^2}{8}-\frac{1}{2}\ln^2(\sqrt{2}+1)\]

\[ I=\ln^2(\sqrt{2}+1)-\frac{\pi^2}{4} \]

____________________________________________________________________________

以上是tian_275461的解法,在stackexchange上看到sos440表示

\[\int_{0}^{\pi} \frac{x \cos x}{1+\sin^2 x} \, dx = 4 \chi_{2}(1-\sqrt{2})\]

where

\[\chi_{2}(z) = \sum_{n=0}^{\infty} \frac{z^{2n+1}}{(2n+1)^2}\]

is the Legendre chi function of order 2. By exploiting some identities involving dilogarithm, we find that

\[\chi_{2}(1-\sqrt{2}) = \frac{1}{4} \log^2 (1+\sqrt{2}) - \frac{3}{8}\zeta(2).\]

This gives the answer

\[\int_{0}^{\pi} \frac{x \cos x}{1+\sin^2 x} \, dx = \log^2(1+\sqrt{2}) - \frac{\pi^2}{4}.\]

______________________________________________________________________________

reference:

http://math.stackexchange.com/questions/323603/integrate-displaystyle-int-0-pi-fracx-cosx1-sin2xdx

http://sos440.tistory.com/83

 

Jun 4

       多重对数函数(polylogarithm)也称:Jonquière's function 是数学中一种特殊的幂级数,定义为:
\[ \text{Li}_{s}(z)=\sum_{k=1}^{\infty}{\frac{z^{k}}{k^{s}}} \]
一般来说,多重对数函数不像对数函数那样是一个初等函数。上述定义中,自变量$|z| < 1$,$s$对所有复数值有效。通过解析延拓,可以将$z$的定义域扩展到更大的范围。$s=1$时的多重对数函数可以用自然对数表示$\text{Li}_{1}(z)=-\ln{(1-z)}$,$s = 2$和$3$的多重对数函数分别称为 Dilogarithm 及 Trilogarithm,其名称的由来是多重对数函数表示为以下的递推积分式
\[ \text{Li}_{s+1}(z)=\int_{0}^{z}{\frac{\text{Li}_{s}(t)}{t}dt} \]
其中 Dilogarithm 可以表示成
\[ \text{Li}_{2}(z)=\sum_{k=1}^{\infty}{\frac{z^{k}}{k^2}}=-\int_{0}^{z}{\frac{\ln(1-t)}{t}dt}\]

http://zh.wikipedia.org/wiki/%E5%A4%9A%E9%87%8D%E5%AF%B9%E6%95%B0%E5%87%BD%E6%95%B0

http://mathworld.wolfram.com/Dilogarithm.html

http://mathworld.wolfram.com/Polylogarithm.html


有以下4条常用性质值得注意
(1)
\[ \text{Li}_{2}(x)+\text{Li}_{2}(-x)=\frac{1}{2}\text{Li}_{2}(x^2) \]
(2)
\[ \text{Li}_{2}(1-x)+\text{Li}_{2}\left(1-\frac{1}{x}\right)=-\frac{1}{2}\ln^2{x} \]
(3)
\[ \text{Li}_{2}(x)+\text{Li}_{2}(1-x)=\frac{1}{6}\pi^2-\ln{x}\cdot\ln(1-x) \]
(4)
\[ \text{Li}_{2}(-x)-\text{Li}_{2}(1-x)+\frac{1}{2}\text{Li}_{2}(1-x^2)=-\frac{1}{12}\pi^2-\ln{x}\cdot\ln{(x+1)}\]
证明
(1)
\begin{align*}
   \text{Li}_{2}(x)+\text{Li}_{2}(-x)&=\sum_{n=1}^{\infty}{\frac{x^n+(-x)^n}{n^2}}\\
   &=\sum_{k=1}^{\infty}{2\cdot\frac{x^{2k}}{(2k)^2}}\\
   &=\sum_{k=1}^{\infty}{\frac{x^{2k}}{2k^2}}=\frac{1}{2}\text{Li}_{2}(x^2)
\end{align*}
(2)

\[ \int_{0}^{1-x}{\frac{-\ln{(1-t)}}{t}dt}+\int_{0}^{1-\frac{1}{x}}{\frac{-\ln{(1-t)}}{t}dt}=I_{1}+I_{2} \]
对$I_{2}$作替换$\displaystyle t=1-\frac{1}{1-y}\Rightarrow dt=-\frac{1}{(1-y)^2}dy$
\[ I_{2}=\int_{0}^{1-x}{\frac{\ln(1-y)}{y(1-y)}dy}=\int_{0}^{1-x}{\frac{\ln(1-y)}{y}dy}+\int_{0}^{1-x}{\frac{\ln(1-y)}{1-y}dy} \]
所以
\[ I_{1}+I_{2}= \int_{0}^{1-x}{\frac{\ln(1-y)}{1-y}dy}=-\frac{1}{2}\ln^{2}(1-y)\bigg|_{0}^{1-x}=-\frac{1}{2}\ln^2{x} \]
(3)

\[ \int_{0}^{x}{\frac{-\ln(1-t)}{t}dt}+\int_{0}^{1-x}{\frac{-\ln(1-t)}{t}dt}=I_{1}+I_{2} \]
对$I_{1}$作替换$y=1-t $
\begin{align*}
I_{1}+I_{2}&=\int_{1-x}^{1}{\frac{-\ln{y}}{1-y}dy}+\int_{0}^{1-x}{\frac{-\ln(1-t)}{t}dt}\\
&=-\ln{x}\cdot\ln{(1-x)}+\int_{0}^{1}{\frac{-\ln(1-t)}{t}dt}\\
&=-\ln{x}\cdot\ln{(1-x)}+\sum_{n=1}^{\infty}{\frac{1}{n^2}}\\
&=\frac{\pi^2}{6}-\ln{x}\cdot\ln(1-x)
\end{align*}
(4)
记\[ A=\text{Li}_{2}(-x)-\text{Li}_{2}(1-x)+\frac{1}{2}\text{Li}_{2}(1-x^2)\]
\begin{align*}
A+\text{Li}_{2}(x)+\text{Li}_{2}(1-x)&=\frac{1}{2}\left(\text{Li}_{2}(x^2)+\text{Li}_{2}(1-x^2)\right)\\ 
&=\frac{1}{2}\cdot\left(\frac{1}{6}\pi^2-\ln{x^2}\cdot\ln(1-x^2)\right)\\
&=\frac{1}{12}\pi^2-\ln{x}\cdot\ln(1-x^2)
\end{align*}
马上得到
\[ A=-\frac{1}{12}\pi^2-\ln{x}\cdot\ln(1+x) \]

——————————————————————————————————————————————————————

相关的特殊值\begin{align*}
\text{Li}_{2}(0)&=0\\
\text{Li}_{2}(1)&=\frac{1}{6}\pi^2\\
\text{Li}_{2}(-1)&=-\frac{1}{12}\pi^2\\
\text{Li}_{2}\left(\frac{1}{2}\right)&=\frac{1}{12}\pi^2-\frac{1}{2}\ln^2{2}\\
\text{Li}_{2}(-\phi)&=-\frac{1}{10}\pi^2-\ln^2{\phi}\\
\text{Li}_{2}\left(-\frac{1}{\phi}\right)&=-\frac{1}{15}\pi^2+\frac{1}{2}\ln^2{\phi}\\
\text{Li}_{2}\left(-\frac{1}{\phi^{2}}\right)&=\frac{1}{15}\pi^2-\ln^2{\phi}\\
\text{Li}_{2}\left(\frac{1}{\phi}\right)&=\frac{1}{10}\pi^2-\ln^2{\phi}\\
\end{align*}
这里$\displaystyle \phi=\frac{\sqrt{5}+1}{2}$是Golden ratio。

_______________________________________________________________________________________________________________

附上搜集到的一本好书《Polylogarithms and Associated Functions》 by Lewin

http://pan.baidu.com/share/link?shareid=2376620720&uk=3456571357